explanation would really help​

Explanation Would Really Help

Answers

Answer 1

Answer:

35

Step-by-step explanation:

the whole line is 49 and line DB is 30.

minus 16 from 30 so 30-16=14

then you minus line CB from EB so 49-14=35

that makes line EC 35


Related Questions

VERY URGENT
The sum of two numbers is 36. The smaller number is 18 less than the larger number. What are the numbers?
Larger number:
Smaller number:

Answers

Answer:

Larger number: 22

The smaller number: 14

HOPE THIS HELPS

Inequality of y<-4+3 on graph

Answers

Answer:

[tex]y < - 1[/tex]

Step-by-step explanation:

[tex]y < - 4 + 3[/tex]

[tex]y < - 1[/tex]

Hope it is helpful...

[tex] \sf \: y < - 4 + 3 \\ \sf \: y < - 1[/tex]

[tex] \sf \: Just \: add \: - 4 \: and \: 3 \: and \: you \: \\ \sf will \: get \: the \: inequality \: in \: the \: simplest \: form.[/tex]

Write the product in simplest form: 1 2/3 x 3 4/5=

Answers

Answer:

19/3

Step-by-step explanation:

1 2/3 = (1*3 + 2)/3 = 5/3

3 4/5 = (5*3 + 4)/5 = (15 + 4)/5 = 19/5

5/3 * 19/5 = 19/3    The 5s cancel.

The simplest form is not obvious to me. I think the answer above is in simplest form, and I will put that below the answer above. But 6 1/3 is also possible.

Answer:

4 1/3

Step-by-step explanation:

1 [tex]\frac{2}{3}[/tex] × 3 [tex]\frac{4}{5}[/tex] =

[tex]\frac{5}{3}[/tex] × [tex]\frac{19}{5}[/tex] =

[tex]\frac{95}{15}[/tex] = 4 [tex]\frac{5}{15}[/tex] = 4 1/3

Find the volume of this figure

Answers

Answer:

A

Step-by-step explanation:

36*12*6+12*24*3=3456

Answer:

3456

Step-by-step explanation:

36*12*6 + 24*12*3

just add up the two volumes

Help me out thankssssss !!!!!!

Answers

Answer:

78/2=39°

Step-by-step explanation:

thx for the points

you spin the spinner once
What is P(5)?

Answers

0-10

A spinner is split into 11 different pieces so you are less likely to land on five than the other numbers.

The probability of getting 5 on spinner is 0.25.

What is the probability?

Probability can be defined as the ratio of the number of favourable outcomes to the total number of outcomes of an event.

We know that, probability of an event = Number of favourable outcomes/Total number of outcomes.

Given that, on the spinner there are four numbers {3, 4, 5, 6}.

Number of favorable outcomes = 1

Total number of outcomes = 4

So, P(5) = 1/4

= 0.25

Therefore, the probability of getting 5 on spinner is 0.25.

To learn more about the probability visit:

https://brainly.com/question/11234923.

#SPJ2

suppose a charity received a donation of $26.2 million. if this represents 49% of the charity's donated funds, what is the total amount of its donated funds? round your answer to the nearest million​

Answers

Answer:

53 million.

Step-by-step explanation:

$26.2 / total = 0.49 ($ in millions)

Then total = (26.2) ÷ 0.49 = 53.47 millions which we can round up to 53 million $

Someone help me please

Answers

9514 1404 393

Answer:

  $448

Step-by-step explanation:

The given formula will tell you the amount of interest earned:

  I = Prt

  I = $400·0.02·6 = $48

The account balance will be the sum of the original deposit and the interest it earns:

  $400 +48 = $448 . . . . balance after 6 years



Match each system of equations to the inverse of its coefficient matrix, A^-1and the matrix of its solution, X.

Answers

Answer: Hello the system of equations and other data required are missing attached below are the missing data

answer :

attached below

Step-by-step explanation:

1st system of equations ( top left ) = Image number 8

system of equation ( Image 10 ) = Image 4

system of equation ( Image 9 ) = Image 6

system of equations ( Image 7 ) = Image 2

attached below is a pictorial representation of the solution

Please help me figure out which is the correct answer, i have attached the picture for you to look at. Thanks so much!

Answers

Answer:

The answer is B ;)

Step-by-step explanation:

4x+5 is greater than or equal to 13

subtract 5 from each side to get 4x is greater than or equal to 8

then divide both sides by 2 to get

x is greater than or equal to 2

Which of the following is the product of 7/8 and -4/21?  a.- 1/6 b.1/12 c.- 16/21 d.- 147/32​

Answers

Answer:

A. -1/6

Step-by-step explanation:

7/8 × -4/217 can be divided by 21 wich will make the fraction -4/21 turn to -4/3 and the fraction 7/8 turn to 1/8-4 can be divided by eight which will make the fraction 1/8 turn to 1/2 and the fraction -4/3 will turn to -1/31/2 × -1/3 = -1/6

The product of 7/8 and -4/21 is a.- 1/6.

What is the product?

In mathematics, a product is the result of multiplication, or an expression that identifies factors to be multiplied.

Now the given numbers are,

7/8 and -4/21

now taking 7/8 and since 8= 4*2

so, 7/8 = 7/(4*2)

Again taking -4/28 and since 28 = 7*4

so, -4/21 = -4/(7*3)

hence the product of 7/8 and -4/21 is given as,

7/8*(-4/21) = 7/(4*2) x -4/(7*3)

taking alike terms together we get,

7/8*(-4/28) = (7/7)(4/4)(-1/2*3)

⇒ 7/8*(-4/28) = -1/6

Hence the correct option is a.-1/6

Therefore,The product of 7/8 and -4/21 is a.- 1/6.

Learn more about product :

https://brainly.com/question/1549083

#SPJ6

Soosie's Cookie Company had fixed costs of $1250 and variable costs of $4.25 per dozen gourmet cookies that were baked and packaged for sale. Write an equation that can be used to determine the total cost when x dozens of cookies are baked and sold. Then determine the total cost of baking and selling 85 dozen gourmet cookies.

Answers

Answer:

$1250.00 + (x)$4.25 =

Step-by-step explanation

So fixed costs are $1,250.00.....every dozen you bake add $4.25 to the costs, cook one dozen you have $1,250.00 plus $4.25 or a total of $1,254.25.

Cook 10 dozen and you add $45.25 = the $1,250.00 for a new total of $1,295.25.

Find the distance between the pair of points: (2,6) and (0,−3).

Answers

Answer:

d = √85

Step-by-step explanation:

d^2 = (X2 - X1)^2 + (Y2 - Y1)^2

= (2 - 0)^2 + (6 + 3)^2

= 4 + 81

d = √85

a piece of ribbon is 5.4 meters long. What will be the length of 8 similar pieces?​

Answers

Answer:

43.2 meters

Step-by-step explanation:

Multiply the length of one piece of ribbon by 8 to determine the total length

5.4 * 8 =43.2 meters

5.4x8=43.2
answer: 43.2 meters

Find the values of x,y and z

Answers

Step-by-step explanation:

x = 180-106 = 74°

y = x/2 = 74/2 = 37°

4z+6 =106

4z = 100

z = 25°

you and your friend start biking to opposite directions from the same point. You travel 108 feet every 8 seconds. Your friend travels 63 feet every 6 seconds. a) how far apart are you and your friend after 15 minutes? b) after 20 minutes you take a 5- minutes rest, but your friend does not. How far apart are you and your friend after 40 minutes? Explain your reasoning.

Answers

Answer:

To find your distance apart, you can convert 15 minutes to seconds because that is what the rate is given in.

15 mins x 60 seconds = 900 seconds.

Step-by-step explanation:

In 900 seconds, there are 900/8=112.5 groups of 8 seconds. This means there are 112.5 groups of 108 feet.

112.5 x 108 = 12150 feet

12150 feet/5280 feet is approximately 2.3 miles.

For your friend you take 900/6 = 150 groups of 6 seconds.

150 x 63 = 9540 feet

9540 feet/5280 feet = 1.79 miles

1.79 miles + 2.3 miles = 2.09 miles

You and your friend are about 2.09 miles apart after 15 minutes.

B). If you go 2.3 miles every 15 minutes, that means you travel about 0.77 miles every 5 minutes. If you travel 20 more minutes (40-15-5) that would be 0.77x 4=3.07 miles more

3.07 + 2.3 = 5.37 miles after 40 minutes (you).

Your friend travels 1.79 in 15 minutes, so 1.79/3 = 0.6 miles every 5 minutes

0.6 x 5 (5 x 5 = 25 minutes)= 3 miles

1.79 + 3 miles= 4.79 miles (friend)

4.79 + 5.37 = 10.16 miles

You and your friend are about 10.16 miles apart after 40 minutes.

How many liters is 350 milliliters?

A. 3500 L
B. 3.50 L
C. 0.350 L
D. 0.035 L

Answers

Answer: C

Step-by-step explanation:

Step-by-step explanation:

1 L equals to 1000 mL

so 350 mL equals to 0.35L

The answer is C

Use the Squeeze Theorem ​

Answers

Answer:

See Below.

Step-by-step explanation:

We want to use the Squeeze Theorem to show that:

[tex]\displaystyle \lim_{x \to 0}\left(x^2\sin\left(\frac{2}{x}\right)\right)=0[/tex]

Recall that according to the Squeeze Theorem, if:

[tex]\displaystyle g(x)\leq f(x) \leq h(x)[/tex]

And:

[tex]\displaystyle \lim_{x\to c}g(x) =\lim_{x\to c}h(x) = L[/tex]

Then:

[tex]\displaystyle \lim_{x\to c}f(x)=L[/tex]

Recall that the value of sine is always ≥ -1 and ≤ 1. Hence:

[tex]\displaystyle -1 \leq \sin\left(\frac{2}{x}\right) \leq 1[/tex]

We can multiply both sides by x². Since this value is always positive, we do not need to change the signs. Hence:

[tex]\displaystyle -x^2\leq x^2\sin\left(\frac{2}{x}\right)\leq x^2[/tex]

Let g = -x², h = x², and f = x²sin(2 / x). We can see that:

[tex]\displaystyle \lim_{x \to 0}g(x) = \lim_{ x \to 0}h(x) = 0[/tex]

And since g(x) ≤ f(x) ≤ h(x), we can conclude using the Squeeze Theorem that:

[tex]\displaystyle \lim_{x \to 0}f(x) = \lim_{x \to 0}x^2\sin\left(\frac{2}{x}\right)=0[/tex]

X/30 + X/40= 1
Solve for x

Answers

Answer:

17 1/7 I think

Step-by-step explanation:

x=17.1

Step-by-step explanation:

[tex] \frac{x}{30} + \frac{x}{40} = 1 \\ \frac{3x + 4x}{120} = 1 \\ \frac{7x}{120} = 1 \\ \frac{7x}{120} = 1 \\ 7x = 120 \\ x = \frac{120}{7} \\ x =17.1 [/tex]

Cannot seem to figure this one out.

Answers

Answer:

cosB = 2√6 / 7

Step-by-step explanation:

use the pythagorean theorem to find the missing side

a² + b² = c²

a² + 5² = 7²

a² + 25 = 49

a² = 24

a = √24

a = 2√6

--------------------------

cosB = adj/hyp

cosB = 2√6 / 7

Question 2.1
Solve the following problem. Round to one decimal place if
necessary. If your answer is correct you will see an image
appear on your screen.

Answers

Answer:

x = 5.3

Step-by-step explanation:

Reference angle (θ) = 24°

Opposite side to the reference angle = x

Adjacent side = 12

Apply TOA, which is:

Tan θ = Opp/Adj

Plug in the values

Tan 24° = x/12

12*Tan 24° = x

5.34274422 = x

x = 5.3 (approximated to one decimal place)

A standard deck of 52 cards has 4 suits (spades, clubs, hearts, and diamonds) with 13 different cards (ace, 2, 3, 4, 5, 6, 7, 8, 9, 10, jack, queen, king) in each suit. If you are dealt exactly two cards from the deck without replacement, what is the probability that you are dealt a pair (matching cards in different suits)

Answers

Answer:

P(a pair with matching cards in different suits) = 1/52

Step-by-step explanation:

We are told that there are 4 suites and each suit has 13 different cards. This is a total of 52 cards.

Thus;

Probability of selecting one card of a particular suit = 13/52 = 1/4

If we now want to select a matching card of another suit without replacing the first one, then, we now have; 52 - 13 = 39 cards. Now, there are only 3 matching cards of the 3 remaining suits that is same as the first card drawn.

Thus; probability = 3/39 = 1/13

Thus;

P(a pair with matching cards in different suits) = 1/4 × 1/13

P(a pair with matching cards in different suits) = 1/52

Suppose that E and F are points on the number line.
If EF=9 and E lies at 4, where could F be located?
If there are several locations, separate them with commas.

Answers

9514 1404 393

Answer:

  -5 or 13

Step-by-step explanation:

A point that is 9 units from 4 on the number line ca be located at ...

  4 - 9 = -5

  4 + 9 = 13

F could be either of -5, 13.

Answer a, b and c. See image below

Answers

Answer:

a) 3/5 < 4/5

b) In general if two fractions have the same denominator, then whichever fraction has the numerator closer to its denominator will be the largest fraction.

c)  [tex]\frac{7}{10} > \frac{9}{15}[/tex]  or  [tex]\frac{7}{10}<\frac{9}{15}[/tex]

Step-by-step explanation:

a) 3/5 < 4/5

Flip the sign and the placement of the fraction so 3/5 is less then 4/5.

b) In general if two fractions have the same denominator, then whichever fraction has the numerator closer to its denominator will be the largest fraction.

c) We need to change the denominators to a common denominator to compare the size of the two fractions:

[tex]\frac{7}{10}[/tex] × [tex]\frac{3}{3}[/tex] = [tex]\frac{21}{30}[/tex]

[tex]\frac{9}{15}[/tex] ×  [tex]\frac{2}{2}[/tex] = [tex]\frac{18}{30}[/tex]

The common denominators of the two fractions is 30. Comparing the two fractions:

[tex]\frac{21}{30} >\frac{18}{30}[/tex]  or  [tex]\frac{18}{30}<\frac{21}{30}[/tex]

so we get:  [tex]\frac{7}{10} > \frac{9}{15}[/tex]  or  [tex]\frac{7}{10}<\frac{9}{15}[/tex]

Question 21
Find the volume.

Answers

Answer:

C

4712 cm³ exactly

Step-by-step explanation:

10² * pi is the area of the circle

then just multiply with the height

Answer:

C. 1500π ≈ 4710 cm³

Step-by-step explanation:

Volume = πr²h

Volume = π * 100 * 15

Volume = 1500π ≈ 4710 cm³

If my answer is incorrect, pls correct me!

If you like my answer and explanation, mark me as brainliest!

-Chetan K

Can someone help me again with these math work?

Answers

Answer:

it look like its 4 and the 4 over 4/5 I think

Jake has a 1 in 5 chance of winning the egg and spoon race.
what is the probability that he will NOT win the race
8%
20%
60%
80%

Answers

Answer:

Step-by-step explanation:

1 in 5 = 1/5 = 20% that he wins

Probability that he will NOT win

= 1 - 20%

= 100% -20%

= 80%

Answer:

80%

Step-by-step explanation:

percentage of winning

1/5×100%=20%

percentage of not winning

100%_20%=80%

SOMEONE SMART PLEASE HELP!! If the area of a triangle is 32 yd^2 and the base is 6.4 yds long, find the height​

Answers

Answer:

Height = 10 yd

Step-by-step explanation:

GIVEN :-

Area of the triangle = 32 yd²Base of the triangle = 6.4 yd

TO FIND :-

The height of the triangle

GENERAL FORMULAE TO BE USED IN THIS QUESTION :-

For a triangle with base 'b' & height 'h' , its area = [tex]\frac{1}{2} \times b \times h[/tex]

SOLUTION :-

[tex]Area = \frac{1}{2} \times Base \times Height[/tex]

[tex]=> 32 = \frac{6.4h}{2}[/tex]

[tex]=> 6.4h = 32 \times 2 = 64[/tex]

[tex]=> h = \frac{64}{6.4} = 10\: yd[/tex]

Miguel went shopping for a new phone. Sales tax where he lives is 4%. What number should he multiply the price of the phone by to find the total plus tax in one step?

Answers

Answer:

miges fue y no me compro el celu

Step-by-step explanation:

mal migel

por culpa de brailyn reverg

PLEASE HELPPPP ASAP PLS WILL MARK BRAINLIEST

Answers

Answer:

5X+6+3X-2 =180

8X+4. =180

8X. =180-4

8X. =176

X=22

CBD=EBF=3x-2

=66-2

=64

CBD=64

Other Questions
Hownis the total magnification of a cell calculated Solve for in the diagram below.(2x + 457RO7 - Why was Cambdia dragged into the Vietnam war? Write a three-sentence summary of Gold Rush! using at least one word from the spelling list in each sentence. A school sold a total of 250 tickets to a show. Adult tickets cost $7.25 and student tickets cost $5.50. At the end of the evening the amount of moneycollected was $1,602.50. Write and solve a system of equations to determine the number of student tickets sold 7.25x+5 5y=1,602.501 / 10000 Word Limit is it possible to make a triangle with a measure of 1o m 6m and 7m Which is an equation of a direct proportion? CLEAR CHECK y=8x y=12x+4 y=12x y=4x4 In early cultures, it was common to have larger families because _____. Select all that apply. there was no need to send the children to schoolparents did not practice good hygienechildren were needed to run the family farm or businessmany children died in childhood . A circuit is set up with two parallel resistors, each of a resistance of 250. a. What is the total resistance for this circuit? Radiation exerts pressure on surfaces on which it lalls (radintion pressure). Will this pressure be greater on a shiny surface or a dark surface What is the area of a triangle with a base of 7cm and a height of 120% of the base?Calculate the area Gandhi *O Non-violent Indian independence leaderO Totalitarian leader of the Soviet UnionO Leader of Chinese CommunistsO None of the above Which of the following best describes the graph of the polynomial functionbelow? Take a look at my hair...I use my hair to express myself... which of the following ideas is central to communist philosiohy? There are two aircraft carriers, A and B, and carrier A is longer in length than the carrier B.The total length of these two carriers is 4198 feet, while the difference of their lengths is only10 feet. the question= find the percent of each number 1.) 78% of 160 2.) 12% of 325 also could u put an explanation I WILL GIVE YOU BRAINLYESTWe have learned that the universe is expanding. Is it expanding at some particular point? Exactly what is expanding? Write a detailed report.Find the surface area or the approximate surface area of each figure. Question 1 of 10What is the x-intercept of the function graphed below? Help!! Please also explain how you got your answer